Difference between revisions of "1986 AJHSME Problems/Problem 2"

m (Solution)
m
Line 1: Line 1:
 
==Problem==
 
==Problem==
  
Which of the following numbers has the largest reciprocal?
+
Which of the following numbers has the largest [[reciprocal]]?
  
 
<math>\text{(A)}\ \frac{1}{3} \qquad \text{(B)}\ \frac{2}{5} \qquad \text{(C)}\ 1 \qquad \text{(D)}\ 5 \qquad \text{(E)}\ 1986</math>
 
<math>\text{(A)}\ \frac{1}{3} \qquad \text{(B)}\ \frac{2}{5} \qquad \text{(C)}\ 1 \qquad \text{(D)}\ 5 \qquad \text{(E)}\ 1986</math>
Line 7: Line 7:
 
==Solution==
 
==Solution==
  
For positive numbers, the larger the number, the smaller it's reciprocal. Likewise, smaller numbers have larger reciprocals.
+
For [[positive|positive numbers]], the larger the number, the smaller its reciprocal. Likewise, smaller numbers have larger reciprocals.
  
 
Thus, all we have to do is find the smallest number.
 
Thus, all we have to do is find the smallest number.
Line 18: Line 18:
 
==See Also==
 
==See Also==
  
[[1986 AJHSME Problems]]
+
{{AJHSME box|year=1986|num-b=1|num-a=3}}
 +
[[Category:Introductory Algebra Problems]]

Revision as of 18:44, 19 May 2009

Problem

Which of the following numbers has the largest reciprocal?

$\text{(A)}\ \frac{1}{3} \qquad \text{(B)}\ \frac{2}{5} \qquad \text{(C)}\ 1 \qquad \text{(D)}\ 5 \qquad \text{(E)}\ 1986$

Solution

For positive numbers, the larger the number, the smaller its reciprocal. Likewise, smaller numbers have larger reciprocals.

Thus, all we have to do is find the smallest number.

But which one is it? $\frac{1}{3}$? or $\frac{2}{5}$? We see that $\frac{1}{3} = \frac{5}{15}$, and $\frac{2}{5} = \frac{6}{15}$, so obviously $\frac{1}{3}$ is smaller.

$\boxed{\text{A}}$

See Also

1986 AJHSME (ProblemsAnswer KeyResources)
Preceded by
Problem 1
Followed by
Problem 3
1 2 3 4 5 6 7 8 9 10 11 12 13 14 15 16 17 18 19 20 21 22 23 24 25
All AJHSME/AMC 8 Problems and Solutions